Howdy, Stranger!

It looks like you're new here. If you want to get involved, click one of these buttons!

Parallel Method of Reasoning & Contrapositives

yeghidyeghid Member

I just have one specific question. On parallel reasoning questions, if an answer choice essentially has the same logical force, # terms, and structure as the stimulus except its conclusion is the contrapositive of the previous terms, does that disqualify it as a right answer?

For instance:

Stimulus:

A --> B;
B --> C;
Thus, A --> C

Answer choice:

D --> E
E --> F
Thus, F --> D?

Comments

  • CSieck3507CSieck3507 Member
    1376 karma

    No it does not because the logical structure is the same. So it would be correct. However, if you have an answer choice that had the correct D->F you would have to pick that AC. But, the lsat wouldnt do that because there cant be two correct AC's.

  • yeghidyeghid Member
    6 karma

    Thank you so much for the tip!

Sign In or Register to comment.